bill's blog

Writeup: IMO 2004 SL A2

Hey everyone! I haven't posted here in a while because I've been busy with finals and starting my summer research, but I'm glad I finally have time to sit down and revive this blog again. Here's my solution for A2 on the 2004 IMO Shortlist.

Problem Statement:

Let a0, a1, a2, ... be an infinite sequence of integers satisfying the equation an=|an+1an+2| for all n0, where a0 and a1 are two different positive integers.

Can this sequence a0, a1, a2, ... be bounded?

My Solution:

First, a couple facts to note:

  1. All ai must be nonnegative, since they are the absolute value of a number.
  2. We can either have an+2=an+an+1, or an+2=an+1an. From the first fact (above), if anan+1, we must have an+2=an+an+1. The latter equivalence makes an+2 negative.
  3. It is impossible to have ai=0, and it is also impossible to have ai=ai+1. What follows is the proof of this fact.

Proof:

First, we prove it's impossible to have ai=ai+1. For the sake of contradiction, assume there exists i such that ai=ai+1. Then there exists minimal i with ai=ai+1. We can assume i3 since a0a1, a1a2 since |a1a2|=a00, and a3a2 since |a3a2|=a10.

Since i3, this would imply that ai1=0, which would imply that ai3=|ai2ai1|=ai2 since ai2 is positive, which would imply the existence of a smaller j=i3 such that aj=aj+1. Contradiction!

Now, we proceed with the proof. Let D={i | ai>ai+1}. Note that if |D| is finite, then N such that nN, an<an+1. If an<an+1<an+2, then we must have an+2=an+1+an for all nN, which obviously diverges and is not bounded.

Now, assume that |D| is instead infinite. Number the i in |D| from smallest to largest as ik. Then note that ik+1ik2 since if aik>aik+1, then aik+2=aik+1+aik>aik+1.

Now note that aik is a strictly increasing quantity.

Proof: Note that j such that ik<j<ik+1, we must have aj<aj+1. Note, also that aik+2=aik+aik+1>aik. This implies that aik+1>aik. Since the integers are closed under addition, this means that we must have aik+1aik+1, so aik must be unbounded and therefore a is unbounded.